You are on page 1of 28

RULE 114 BAIL TOPIC:Section 1. Bail Defined CASES: PADERANGA VS.

COURT OF APPEALS 247 SCRA 741

Facts: Petitioner Miguel Paderanga was belatedly charged as co-conspirator in the crime of multiple murder for the killing of members of the Bucag family in Gingoog City of which, petitioner was the mayor at the time. The trial of the case was all set to start with the issuance of an arrest warrant for petitioners apprehension, but before it could be served on him, petitioner, through his counsel filed a motion for admission to bail with the trial court. Petitioner furnished copies of the motion to State Prosecutor Gingoyon , to the Regional State prosecutor and the private prosecutor. The trial proceeded to hear the application for bail, but only assistant prosecutor appeared for the prosecution and four of petitioners counsel. Petitioner was then confined at the Cagayan Capitol College General Hospital due to acute costochondritis, his counsel manifested that they were submitting custody over the person of their client to the local chapter of the IBP and that, for purposes of said hearing of his bail application, he is considered being in custody of the law. The prosecution neither supported nor opposed the said application for bail. The trial court admitted petitioner to bail. The following day, petitioner managed to personally appear before the clerk of court of the trial court and posted bail in the amount fixed. He was thereafter arraigned and in the trial that ensued, he also appeared personally and attended all the scheduled court hearings of the case. 20 days after the resolution of the trial court, Prosecutor Gingoyon filed a motion for reconsideration alleging that he received a copy of the petition for admission to bail only a day after the hearing. The trial court denied the motion for reconsideration, and so Prosecutor Gingoyon elevated the matter the respondent Court of Appeals. The Court of Appeals annulled the decision of the trial court in granting bail to petitioner on the ground that they were tainted with grave abuse of discretion. Responednt court observed that at the time of petitioners application for bail, he was not yet in the custody of the law, apparently because he filed his motion for admission to bail before he was actually arrested or had voluntarily surrendered. Respondent court also noted that petitioner was charged with a crime punishable by reclusion perpetua, the evidence of guilt was strong as borne out by the fact that no bail was recommended by the prosecution, for which reasons held that the grant of bail was doubly improvident. Issue: A. Whether or not petitioner was in the custody of the law when it applied for bail before the arrest warrant was served on him. B. Whether or not bail was granted with grave abuse of discretion considering that petitioner was charged with a crime punishable by reclusion perpetua and the evidence of guilt was strong. Ruling: A. Section 1 of Rule 114, as amended, defines bail as the security given for the release of a person in custody of the law, furnished by him or a bondsman, conditioned upon his appearing before any court as required under the conditions specified in said Rule. Its main

purpose, then, is to relieve an accused from the rigors of imprisonment until his conviction and yet secure his appearance at the trial. 10As bail is intended to obtain or secure one's provisional liberty, the same cannot be posted before custody over him has been acquired by the judicial authorities, either by his lawful arrest or voluntary surrender. 11As this Court has put it in a case "it would be incongruous to grant bail to one who is free." As a paramount requisite then, only those persons who have either been arrested, detained, or other wise deprived of their freedom will ever have occasion to seek the protective mantle extended by the right to bail. The person seeking his provisional release under the auspices of bail need not even wait for a formal complaint or information to be filed against him as it is available to "all persons" where the offense is bailable. The rule is, of course, subject to the condition or limitation that the applicant is in the custody of the law. On the other hand, a person is considered to be in the custody of the law (a) when he is arrested either by virtue of a warrant of arrest issued pursuant to Section 6, Rule 112, or by warrantless arrest under Section 5, Rule 113 in relation to Section 7, Rule 112 of the revised Rules on Criminal Procedure, or (b) when he has voluntarily submitted himself to the jurisdiction of the court by surrendering to the proper authorities. In the case of herein petitioner, it may be conceded that he had indeed filed his motion for admission to bail before he was actually and physically placed under arrest. He may, however, at that point and in the factual ambience therefore, be considered as being constructively and legally under custody. Thus in the likewise peculiar circumstance which attended the filing of his bail application with the trail court, for purposes of the hearing thereof he should be deemed to have voluntarily submitted his person to the custody of the law and, necessarily, to the jurisdiction of the trial court which thereafter granted bail as prayed for. In fact, an arrest is made either by actual restraint of the arrestee or merely by his submission to the custody of the person making the arrest. 19The latter mode may be exemplified by the so-called "house arrest" or, in case of military offenders, by being "confined to quarters" or restricted to the military camp area B. Section 13, Article III of the Constitution lays down the rule that before conviction, all indictees shall be allowed bail, except only those charged with offenses punishable by reclusion perpetua when the evidence of guilt is strong. In pursuance thereof, Section 4 of Rule 114, as amended, now provides that all persons in custody shall, before conviction by a regional trial court of an offense not punishable by death, reclusion perpetua or life imprisonment, be admitted to bail as a matter of right. The right to bail, which may be waived considering its personal nature and which, to repeat, arises from the time one is placed in the custody of the law, springs from the presumption of innocence accorded every accused upon whom should not be inflicted incarceration at the outset since after trial he would be entitled to acquittal, unless his guilt be established beyond reasonable doubt. Thus, the general rule is that prior to conviction by the regional trial court of a criminal offense, an accused is entitled to be released on bail as a matter of right, the present exceptions thereto being the instances where the accused is charged with a capital offense or an offense punishable by reclusion perpetua or life imprisonment and the evidence of guilt is strong. Under said general rule, upon proper application for admission to bail, the court having custody of the accused should, as a matter of course, grant the same after a hearing conducted to specifically determine the conditions of the bail in accordance with Section 6 (now, Section 2) of Rule 114. On the other hand, as the grant of bail becomes a matter of judicial discretion on the part of the court under the exceptions to the rule, a hearing, mandatory in nature and which should be summary or otherwise in the discretion of the court, is required with the participation of both the defense and a duly notified representative of the prosecution, this time to ascertain whether or not the evidence of guilt is strong for the provisional liberty of the applicant. Of course, the burden of proof is on the prosecution to show that the evidence meets the required quantum. The Court ruled that Prosecutor Abejo was authorized and could validly represent the prosecution in the hearing for bail. He was also explicitly instructed about the position of the

Regional State Prosecutors Office, to manifest to the court that the prosecution was neither supporting nor opposing the application for bail and that they were submitting the matter to its sound discretion. Obviously, what this meant was that the prosecution, at that particular posture of the case, was waiving the presentation of any countervailing evidence. The prosecution was dispensing with the introduction of evidence en contra and this it did at the proper forum and stage of the proceedings, that is during the mandatory hearing for bail and after the trial court had fully satisfied itself that such was the position of the prosecution.

SERAPIO VS. SANDIGANBAYAN 396 SCRA 443 Facts: Petitioner Edward Serapio was a member of the Board of Trustees and the legal counsel of the Erap Muslim Youth Foundation. Sometime 2000, petitioner received on its behalf a donation in the amount of Php 200M through Chavit Singson. Petitioner received the donation worth the Foundations account. In 2000, Chavit Singson publicly accused President Estrada and his family members and friends of engaging in several illegal activities which triggered the filing with the Office of the Ombudsman several criminal complaints against the petitioner, Joseph Estrada and his son. On April 4, 2001, Ombudsman filed with the Sandiganbayan Informations against the former president, one of which, for plunder. No bail was recommended for the provisional release of all the accused including the petitioner. The case was raffled to a special division which was subsequently created by the Supreme Court. On 25 April 2001, Sandiganbayan issued a resolution finding probable cause to justify the issuance of warrants of arrest for the accused. Arraignment was set on 27 January 2001. In the meantime, petitioner filed with Sandiganbayan an Urgent Petition for bail, which was set for hearing on May 4, 2001. Petitioners co-accused Jinggoy Estrada filed a motion alleging that he was entitle to bail as a matter of right. During the hearing on May 4, 2001 on petitioners Urgent Petition for Bail, the prosecution moved for the resetting of the arraignment of the accused earlier than the June 27 schedule. However, Sandiganbayan denied the motion of the prosecution and issued an order declaring that the petition for bail can and should be heard BEFORE petitioners arraignment on 27 June. On June 1, Sandiganbayan issued a resolution requiring the attendance of petitioner as well as all the other accused during the hearing on the petitioner for bail considering that under Section 8, Rule 115 of the Revised Rules of Court, whatever evidence adduced during the hearing shall be considered automatically reproduced at the trial. The people insist that arraignment is necessary before bail hearings may be commenced because it is only upon arraignment that the issues are joined. The people further stress the it is only when an accused pleads not guilty may he filed a petition for bail and if he pleads guilty, then there would be no need for him to file said petition. It is also the contention of the people that it is only during arraignment that the accused is informed of the precise charge against him. He must then be arraign first prior to bail hearings to prevent him from late on assailing the validity of the bail hearings on the ground that he was not properly informed of the charge considering that under section 8 of Rule 114, evidence presented during bail hearings are reproduce in the trial. Arraignment before bail hearings also diminished the possibility of accuseds flight since trial in absentia may be had only if an accused escapes after he has been arraigned. However, the bail hearing again did not proceed because the petitioner filed with the information a motion to quash the amended information on the grounds that as against him, the amended information does not allege a combination of series of over or criminal acts

constitutive of plunder. According to the prosecution, the motion to quash the amended information was antithetical to his petition for bail. Petitioner also prays for the issuance of habeas corpus. Issues: (a) (b) (c) (d) Ruling: (a) Although the petitioner was already arraigned, no plea has yet been entered thereby rendering the issue of whether an arraignment is necessary before the conduct of bail hearings in the petitioners case moot. Nonetheless, the court held that arraignment of an accused is not a pre-requisite to the conduct of hearings on his petition for bail. A person is allowed to petition for bail as soon as he is deprived of his of his liberty by virtue of his arrest or voluntary surrender. In Lavides vs. CA, the court ruled that in cases where it is authorized, bail should be granted before arraignment otherwise the accused may be precluded from filing a motion to quash. However, this pronouncement should not be taken to mean that the hearing on a petition for bail should at all times precede arraignment, because the rule is that a person deprived of his liberty by virtue of his arrest or voluntary surrender may apply for bail as soon as he is deprived of liberty even before a complaint or information is filed against him. The case of Lavides must be understood in light of the fact that the accused in said case filed a petition for bail as well as a motion to quash. Hence, in that case, the court held that to condition the grant of bail to an accused on his arraignment would be to place him in a position where he had to choose between filing a motion to quash and thus delay his petition for bail and forgoing the filing of the motion to quash so that he can be arraign at once ad therefore be released on bail. Such would undermine the constitutional right of the accused. When a bail is matter of right, an accused may apply for and be granted bail even prior to arraignment. The Lavides case also implies that an application for bail in a case involving an offense punishable by reclusion perpetua to death may also be heard even before an accused is arraigned. Sandiganbayan therefore committed grave abuse of discretion amounting to excess of jurisdiction in ordering the arraignment of petitioner before proceeding with the hearing of his petition for bail. (b) Court there is no inconsistency exists between an application of an accused for bail and his filing of a motion to quash. Bail, is the security given for the release of the person in custody of the law. A motion to quash on the other hand is a mode by which an accused assails the validity of a criminal complaint filed against him for insufficiency on its fact in posit of law. These tow relied have objectives which are not necessarily antithetical to each other. However, it is true that if a motion to quash a criminal complaint or information on the ground that the same does not charge any offense is granted and the case is dismissed and the accused is ordered released, the petition for bail of an accused may become moot and academic. (c) Petitioner argues that a joint bail hearing would negate his right to have his petition for bail resolved in a summary proceeding since said hearing might be converted into a full blown trial. Prosecution on the other hand claims that joint hearings will save the court form W/N petitioner should first be arraigned before hearings of his petition for bail may be conducted. W/N petitioner may file a motion to quash the amended Information during the pendency of his petition for bail. W/N a joint hearing of petition for bail for all the accused is mandatory W/N petitioner should instead be released through a writ of habeas corpus.

having to hear the same witnesses and the parties from presenting the same evidences. There is no provision in the Rules of Court governing the hearings of two or more petitioner for bail filed by different accused or that a petition for bail of an accused be heard simultaneously with the trial of the case against the other accused. The matter should be addressed to the sound discretion of the trial court. In the exercise of its discretion, the Sandiganbayan must take into account not only the convenience of the state, including the prosecution but also that of the petitioner and the witnesses. In the case of Ocampo vs. Bernabe, the court ruled that in a petition or bail hearing, the court is to conduct only a summary hearing, meaning such brief and speedy method of receiving and considering the evidence of guilt as is practicable and consistent with the purpose of the hearing which is early to determine the weight of evidence for purposes of bail. The court does not try the merits or enter into the inquiry as to the weight that ought to be given to the evidence against the accused, nor will it speculate on the outcome of the trial or on what further such evidence as has reference to substantial matters. In the case at bar, the case against former President Estrada is an entirely different matter. For, with the participation of the former president in the hearing of petitioners petition for bail, the proceeding assumes completely different dimension. The proceeding will no longer be summary since the proceedings will be full blown which is antithetical to the nature of a bail hearing. The joinder of the petitioners bail will be prejudicial to the petitioner as it will unduly delay the determination of the issue of the right of petitioner to obtain provisional liberty and seek relief from his court. The Sandiganbayn again committed a grave abuse of discretion in ordering a simultaneous hearing of petitioners petition for bail with the trial of the case against former president. (d) In the case at bar, bail is not matter of rights since the accused is charged with a capital offense, but discretionary upon the court. Under Section 8 of rule 114, there must be a showing that the evidence of guilt against a person charged with a capital offense is not strong for the court to grant him bail., thus, upon an application for bail, by the person charged with a capital offense, a hearing must be conducted where the prosecution has the burden of showing that the evidence of guilt against an accused is strong. When the evidence of guilt is strong, bail becomes a matter of right, which is not so in the case at bar. In exceptional cases, habeas corpus may be granted ny the courts even when the person concerned is detained pursuant to a valid arrest or his voluntary surrender. The writ may be issued where the deprivation of liberty while initially valid under the lad had not later become invalid. However, there is no basis fir the issuance of the writ in the case at bar. The general rule is that the writ does not lie where the person alleged to be restrained of his liberty is in the custody of an officer under process issued by a court which had jurisdiction to issued the same applied, because petitioner is under detention pursuant to the order of arrest. Petitioner in fact voluntarily surrendered himself to the authorities.

COMMENDADOR ET.AL VS. DE VILLA G.R. No. 93177 August 2, 1991 CRUZ, J.:p Topic: Fundamental Principles FACTS: The private respondents are officers of the Armed Forces of the Philippines facing prosecution for their alleged participation in the failed coup d' etat that took place on December 1 to 9, 1989. The charges against them are violation of Articles of War (AW) 67 (Mutiny), AW 96 (Conduct Unbecoming an Officer and a Gentleman) and AW 94 (Various Crimes) in relation to Article 248 of the Revised Penal Code (Murder). Ltc Jacinto Ligot (private respondent) applied for bail, but the application was denied by GCM No.14. He thereupon filed with the Regional Trial Court of Quezon City a petition for certiorari and mandamus with prayer for provisional liberty and a writ of preliminary injunction which consequently issued an order granting provisional liberty to Ligot. The trial court rendered judgment inter alia: Xxx Declaring, that Section 13, Article III of the Constitution granting the right to bail to all persons with the defined exception is applicable and covers all military men facing court-martial proceedings. xxx However, Solicitor general contends that military men are exempt from constitutional coverage on the right to bail. ISSUE: Do private respondents have the right to bail? RULING: None. Supreme Court find that the right to bail invoked by the private respondents has traditionally not been recognized and is not available in the military, as an exception to the general rule embodied in the Bill of Rights. This much was suggested in Arula, where we observed that "the right to a speedy trial is given more emphasis in the military where the right to bail does not exist. The argument that denial from the military of the right to bail would violate the equal protection clause is not acceptable.

This guaranty requires equal treatment only of persons or things similarly situated and does not apply where the subject of the treatment is substantially different from others. The accused officers can complain if they are denied bail and other members of the military are not. But they cannot say they have been discriminated against because they are not allowed the same right that is extended to civilians. TRILLANES IV vs HON PIMENTEL FACTS: Petitioner Trillanes, who is charged and detained for staging a coup d'etat referred to as the Oakwood Incident asked the Court that he be allowed to attend all official functions of the Senate. While he was in detention, he ran and won a seat in the Senate. He filed with the RTC an "Omnibus Motion for Leave of Court to be Allowed to Attend Senate Sessions and Related Requests". The trial court denied all the requests in the Omnibus Motion as well as the motion for reconsideration. Petitioner then filed a petition for certiorari to set aside the two Orders and for prohibition and mandamus to (i) enjoin respondents from banning the Senate staff, resource persons and guests from meeting with him or transacting business with him in his capacity as Senator; and (ii) direct respondents to allow him access to the Senate staff, resource persons and guests and permit him to attend all sessions and official functions of the Senate. He stated that he should be allowed bail because he still enjoys the presumption of innocence since unlike Jalosjos he has not been convicted of the crime charged against him. Further, Jalosjos was charged with statutory rape and acts of lasciviousness, crimes involving moral turpitude, while the petitioner is charged with coup d etat which is commonly regarded as a political offense. He further alleged that unlike Jalosjos who attempted to evade trial, he is not a flight risk since he voluntarily surrendered to the proper authorities and such can be proven by the numerous times he was allowed to travel outside his place of detention. ISSUE: Whether or not the accused should be allowed bail to attend to his official functions in senate since unlike Jalosjos he has not been convicted and still enjoys the presumption of innocence HELD: The petitioners application for bail and for release on recognizance was denied. The constitution provides that: All persons, except those charged with offenses punishable by reclusion perpetua when evidence of guilt is strong, shall, before conviction, be bailable by sufficient sureties or be released on recognizance as may be provided by law. The right to bail shall not be impaired even when the privilege of the writ of habeas corpus is suspended. Excessive bail shall not be required. Within the class of offenses covered by the stated range of imposable penalties, there is clearly no distinction as to the political complexion of or moral turpitude involved in the crime charged. The determination that the evidence of guilt is strong, whether ascertained in a hearing of an application for bail or imported from a trial court's judgment of conviction justifies the detention of an accused as a valid curtailment of his right to provisional liberty. This accentuates the proviso that the denial of the right to bail in such cases is "regardless of the stage of the criminal action." Such justification for confinement with its underlying rationale of public self-defense applies equally to detention prisoners like petitioner or convicted prisoners-appellants like Jalosjos. In People v. Hon. Maceda, the court stated that when a person indicted for an offense is arrested, he is deemed placed under the custody of the law. He is placed in actual restraint of liberty in jail so that he may be bound to answer for the commission of the offense. He must be detained in jail during the pendency of the case against him, unless he is authorized by the court to be released on bail or on recognizance. Let it be stressed that all prisoners

whether under preventive detention or serving final sentence can not practice their profession nor engage in any business or occupation, or hold office, elective or appointive, while in detention. This is a necessary consequence of arrest and detention. These inherent limitations, however, must be taken into account only to the extent that confinement restrains the power of locomotion or actual physical movement. It bears noting that in Jalosjos, which was decided en banc one month after Maceda, the Court recognized that the accused could somehow accomplish legislative results. The trial court thus correctly concluded that the presumption of innocence does not carry with it the full enjoyment of civil and political rights. Petitioner is similarly situated with Jalosjos with respect to the application of the presumption of innocence during the period material to the resolution of their respective motions. The Court in Jalosjos did not mention that the presumption of innocence no longer operates in favor of the accused pending the review on appeal of the judgment of conviction. The rule stands that until a promulgation of final conviction is made, the constitutional mandate of presumption of innocence prevails. His contention that he is not a flight risk for voluntarily surrendering to the proper authorities is contravened by subsequent events. He went past security detail and proceeded from the courtroom to a posh hotel to issue certain statements. The account, dubbed this time as the "Manila Pen Incident," proves that petitioner's argument bites the dust. The risk that he would escape ceased to be neither remote nor nil as, in fact, the cause for foreboding became real. Moreover, circumstances indicating probability of flight find relevance as a factor in ascertaining the reasonable amount of bail and in canceling a discretionary grant of bail. In cases involving non-bailable offenses, what is controlling is the determination of whether the evidence of guilt is strong. Once it is established that it is so, bail shall be denied as it is neither a matter of right nor of discretion. Petitioner cannot find solace in Montano v. Ocampo to buttress his plea for leeway because unlike petitioner, then Senator Justiniano Montano was able to rebut the strong evidence for the prosecution. Notatu dignum is this Court's pronouncement therein that "if denial of bail is authorized in capital cases, it is only on the theory that the proof being strong, the defendant would flee, if he has the opportunity, rather than face the verdict of the jury. Jaloslos succinctly expounds that allowing accused-appellant to attend congressional sessions and committee meetings for five (5) days or more in a week will virtually make him a free man with all the privileges appurtenant to his position. Such an aberrant situation not only elevates accused-appellant's status to that of a special class, it also would be a mockery of the purposes of the correction system. WHEREFORE, the petition is DISMISSED. TOPIC: SECTION 2-9. CASES Manotoc vs. CA (May 30, 1986) FACTS: Ricardo Manotoc Jr. was one of the two principal stockholders of Trans-Insular Management Inc. and the Manotoc Securities Inc. (stock brokerage house). He was in US for a certain time, went home to file a petition with SEC for appointment of a management committee for both businesses. Such was granted. However, pending disposition of a case filed with SEC, the latter requested the Commissioner of Immigration not to clear him for departure. Consequently, a memorandum to this effect was issued. There was a torrens title submitted to and accepted by Manotoc Securities Inc which was suspected to be fake. 6 of its clients filed separate criminal complaints against the

petitioner and Leveriza, President and VP respectively. He was charged with estafa and was allowed by the Court to post bail. Petitioner filed before each trial court motion for permission to leave the country stating his desire to go to US relative to his business transactions and opportunities. Such was opposed by the prosecution and was also denied by the judges. He filed petition for certiorari with CA seeking to annul the prior orders and the SEC communication request denying his leave to travel abroad. According to the petitioner, having been admitted to bail as a matter of right, neither the courts that granted him bail nor SEC, which has no jurisdiction over his liberty, could prevent him from exercising his constitutional right to travel. ISSUE: WON petitioners constitutional right to travel was violated. HELD: NO. The court has power to prohibit person admitted to bail from leaving the country because this is a necessary consequence of the nature and function of a bail bond. The condition imposed upon petitioner to make himself available at all times whenever the court requires his presence operates as a valid restriction on his constitutional right to travel. In case he will be allowed to leave the country without sufficient reason, he may be placed beyond the reach of courts. Furthermore, petitioner failed to satisfy trial court and CA of the urgency of his travel, duration thereof, as well as consent of his surety to the proposed travel. He was not able to show the necessity of his travel abroad. He never indicated that no other person in his behalf could undertake such business transaction. Article 3 Sec6: The liberty of abode and of changing the same shall not be impaired except upon lawful order of the court. According to SC, the order of trial court in releasing petitioner on bail constitutes such lawful order as contemplated by the provision on right to travel. G.R. No. 94284 April 8, 1991 RICARDO C. SILVERIO, petitioner, vs. THE COURT OF APPEALS, HON. BENIGNO G. GAVIOLA, as Judge of the Regional Trial Court of Cebu City, Branch IX, and PEOPLE OF THE PHILIPPINES Facts: This is a Petition for Review on Certiorari under Rule 45 of the Rules of Court praying that the Decision of respondent Court of Appeals in CA-G.R. SP No. 15827, entitled "Ricardo C. Silverio vs. Hon. Benigno C. Gaviola, etc., et al.," dated 31 January 1990, as well as the Resolution of 29 June 1990 denying reconsideration, be set aside. On 14 October 1985, Petitioner was charged with violation of Section 20 (4) of the Revised Securities Act in Criminal Case No. CBU-6304 of the Regional Trial Court of Cebu. In due time, he posted bail for his provisional liberty. On 26 January 1988, or more than two (2) years after the filing of the Information, respondent People of the Philippines filed an Urgent ex parte Motion to cancel the passport of and to issue a hold-departure Order against accused-petitioner on the ground that he had

gone abroad several times without the necessary Court approval resulting in postponements of the arraignment and scheduled hearings. Overruling opposition, the Regional Trial Court, on 4 April 1988, issued an Order directing the Department of Foreign Affairs to cancel Petitioner's passport or to deny his application therefor, and the Commission on Immigration to prevent Petitioner from leaving the country. This order was based primarily on the Trial Court's finding that since the filing of the Information on 14 October 1985, "the accused has not yet been arraigned because he has never appeared in Court on the dates scheduled for his arraignment and there is evidence to show that accused Ricardo C. Silverio, Sr. has left the country and has gone abroad without the knowledge and permission of this Court" (Rollo, p. 45). Petitioner's Motion for Reconsideration was denied on 28 July 1988. Petitioner's Certiorari Petition before the Court of Appeals met a similar fate on 31 January 1990. Hence, this Petition for Review filed on 30 July 1990. ISSUE: Whether or not the right to travel can be impaired upon lawful order of the Court, even on grounds other than the "interest of national security, public safety or public health." RULING: Yes. The bail bond he had posted had been cancelled and Warrants of Arrest had been issued against him by reason, in both instances, of his failure to appear at scheduled arraignments. Warrants of Arrest having been issued against him for violation of the conditions of his bail bond, he should be taken into custody. "Bail is the security given for the release of a person in custody of the law, furnished by him or a bondsman, conditioned upon his appearance before any court when so required by the Court or the Rules. Article III, Section 6 of the 1987 Constitution should by no means be construed as delimiting the inherent power of the Courts to use all means necessary to carry their orders into effect in criminal cases pending before them. When by law jurisdiction is conferred on a Court or judicial officer, all auxillary writs, process and other means necessary to carry it into effect may be employed by such Court or officer Petitioner is facing a criminal charge. He has posted bail but has violated the conditions thereof by failing to appear before the Court when required. Warrants for his arrest have been issued. Those orders and processes would be rendered nugatory if an accused were to be allowed to leave or to remain, at his pleasure, outside the territorial confines of the country. Holding an accused in a criminal case within the reach of the Courts by preventing his departure from the Philippines must be considered as a valid restriction on his right to travel so that he may be dealt with in accordance with law. The offended party in any criminal proceeding is the People of the Philippines . It is to their best interest that criminal prosecutions should run their course and proceed to finality without undue delay, with an accused holding himself amenable at all times to Court Orders and processes. The judgment under review is hereby AFFIRMED. People v. Intermediate Appellate Court 147 SCRA 219 FACTS: As a result of a shooting incident the accused was charged with two counts of assault upon an agent in authority and murder with the use of an illegally possessed firearm. The trial court denied the petition for bail of the accused on the ground that the crimes charged were

capital offenses and the evidence of guilt was strong. The Intermediate Appellate Court granted bail to the accused on the ground that the crimes committed involved homicide and not murder. SC RULING: To determine whether the offense charged is capital, the criterion is the penalty regardless of the attendant circumstances. Otherwise, evidence of the aggravating and mitigating circumstance will have to be considered. There will be a complete trial. This will defeat the purpose of bail. EDWARD S. SERAPIO, petitioner, vs. HONORABLE SANDIGANBAYAN and PEOPLE OF THE PHILIPPINES FACTS; Before the Court are two petitions for certiorari filed by petitioner Edward Serapio, assailing the resolutions of the Third Division of the Sandiganbayan denying his petition for bail. The records show that petitioner was a member of the Board of Trustees and the Legal Counsel of the Erap Muslim Youth Foundation, a non-stock, non-profit foundation established in February 2000 ostensibly for the purpose of providing educational opportunities for the poor and underprivileged but deserving Muslim youth and students, and support to research and advance studies of young Muslim educators and scientists. Sometime in April 2000, petitioner, as trustee of the Foundation, received on its behalf a donation in the amount of Two Hundred Million Pesos (P200 Million) from Ilocos Sur Governor Luis Chavit Singson through the latters assistant Mrs. Yolanda Ricaforte. Petitioner received the donation and turned over the said amount to the Foundations treasurer who later deposited it in the Foundations account with the Equitable PCI Bank. . During the hearing on May 4, 2001 on petitioners Urgent Petition for Bail, the prosecution moved for the resetting of the arraignment of the accused earlier than the June 27, 2001 schedule. However, the Sandiganbayan denied the motion of the prosecution and issued an order declaring that the petition for bail can and should be heard before petitioners arraignment on June 27, 2001 and even before the other accused in Criminal Case No. 26558 filed their respective petitions for bail. Accordingly, the Sandiganbayan set the hearing for the reception of evidence on petitioners petition for bail on May 21 to 25, 2001. ISSUE: Whether or not petitioner should first be arraigned before hearings of his petition for bail may be conducted. HELD: The arraignment of an accused is not a prerequisite to the conduct of hearings on his petition for bail. A person is allowed to petition for bail as soon as he is deprived of his liberty by virtue of his arrest or voluntary surrender (Mendoza vs. CFI of Quezon, 51 SCAD 369). an accused need not wait for his arraignment before filing a petition for bail. In Lavides vs. CA, 324 SCRA 321, it was held that in cases where it is authorized, bail should be granted before arraignment, otherwise the accused may be precluded from filing a motion to quash. This pronouncement should be understood in the light of the fact that the accused in said case filed a petition for bail as well as a motion to quash the informations filed against him. It was explained that to condition the grant of bail to an accused on his arraignment would be to place him in a position where he has to choose between: (1) filing a motion to quash and thus delay his release on bail because until his motion to quash can be resolved, his arraignment cannot be held; and (2) foregoing the filing of a motion to quash so that he can be arraigned at once and thereafter be released on bail. This would undermine his constitutional right not to be put on trial except upon a valid complaint or information sufficient to charge him with a crime and his right to bail. It is therefore not necessary that

an accused be first arraigned before the conduct of hearings on his application for bail. For when bail is a matter of right, an accused may apply for and be granted bail even prior to arraignment . ROLITO GO y TAMBUNTING vs. COURT OF APPEALS 206 SCRA 138 FACTS: An information was filed charging herein petitioner Rolito Go for murder before the Regional Trial Court of Metro Manila. Petitioner voluntarily presented himself together with his two lawyers to the police upon obtaining knowledge of being hunted by the latter. However, he was immediately detained and denied his right of a preliminary investigation unless he executes and sings a waiver of the provisions of Article 125 of the Revised Penal Code. Upon omnibus motion for immediate release on recognizance or on bail and proper preliminary investigation on the ground that his warrantless arrest was unlawful and no preliminary investigation was conducted before the information was filed, which is violative of his rights, the same was granted but later on reversed by the lower court and affirmed by the Court of Appeals. The appellate court in sustaining the decision of the lower court held that petitioners warrantless arrest was valid in view of the fact that the offense was committed, the petitioner was clearly identified and there exists valid information for murder filed against petitioner. Hence, the petitioner filed this present petition for review on certiorari before the Supreme Court. ISSUE/S: The issues assailed in the case at bar are the following: 1. whether or not the warrantless arrest of herein petitioner was lawful, and 2. whether or not petitioner waived his right to preliminary investigation. RULING: The general rule on arrest provides that the same is legitimate if effected with a valid warrant. However, there are instances specifically enumerated under the law when a warrantless arrest may be considered lawful. Despite that, the warrantless arrest of herein petitioner Rolito Go does not fall within the terms of said rule. The police were not present at the time of the commission of the offense, neither do they have personal knowledge on the crime to be committed or has been committed not to mention the fact that petitioner was not a prisoner who has escaped from the penal institution. In view of the above, the allegation of the prosecution that petitioner needs to sign a waiver of the provisions of Article 125 of the Revised Penal Code before a preliminary investigation may be conducted is baseless. In this connection, petitioner has all the right to ask for a preliminary investigation to determine whether is probable cause that a crime has been committed and that petitioner is probably guilty thereof as well as to prevent him from the hassles, anxiety and aggravation brought by a criminal proceeding. This reason of the accused is substantial, which he should not be deprived of. On the other hand, petitioner did not waive his right to have a preliminary investigation contrary to the prosecutors claim. The right to preliminary investigation is deemed waived when the accused fails to invoke it before or at the time of entering a pleas at arraignment. The facts of the case show that petitioner insisted on his right to preliminary investigation before his arraignment and he, through his counsel denied answering questions before the court unless they were afforded the proper preliminary investigation. For the above reasons, the petition was granted and the ruling of the appellate court was set aside and nullified. The Supreme Court however, contrary to petitioners allegation, declared that failure to accord the right to preliminary investigation did not impair the validity of the information charging the latter of the crime of murder. G.R. No. 148571 September 24, 2002 GOVERNMENT OF THE UNITED STATES OF AMERICA, Represented by the Philippine Department of Justice, petitioner, vs.

HON. GUILLERMO PURGANAN, Presiding Judge Regional Trial Court of Manila and MARC JIMENEZ a.k.a. MARCIO BATACAN CRESPO, respondent Facts: Pursuant to the existing RP-US Extradition Treaty, the United States Government requested the extradition of Mark B. Jimenez, also known as Mario Batacan Crespo. Upon receipt of the request, the secretary of foreign affairs (SFA) transmitted them to the secretary of justice (SOJ) for appropriate action. The Government of the United States of America, represented by the Philippine DOJ, filed with the Regional Trial Court the appropriate Petition for Extradition. Before the Regional Trial Court could act on the Petition, Respondent Jimenez filed before it an "Urgent Manifestation/Ex-Parte Motion," which prayed that petitioners application for an arrest warrant be set for hearing. Jimenez sought an alternative prayer: that in case a warrant should issue, he be allowed to post bail in the amount of P100, 000. Thereafter, the Regional Trial Court issued its questioned order, directing the issuance of a warrant for his arrest and fixing bail for his temporary liberty at one million pesos in cash. After he had surrendered his passport and posted the required cash bond, Jimenez was granted provisional liberty. Hence, this Petition. Essentially, the Petition prays for the lifting of the bail Order, the cancellation of the bond, and the taking of Jimenez into legal custody. Issue: Whether he is entitled to bail and to provisional liberty while the extradition proceedings are pending. Ruling: Article III, Section 13 of the Constitution, is worded as follows: Art. III, Sec. 13. All persons, except those charged with offenses punishable by reclusion perpetua when evidence of guilt is strong, shall, before conviction, be bailable by sufficient sureties, or be released on recognizance as may be provided by law. The right to bail shall not be impaired even when the privilege of the writ of habeas corpus is suspended. Excessive bail shall not be required." As suggested by the use of the word "conviction," the constitutional provision on bail quoted above, as well as Section 4 of Rule 114 of the Rules of Court, applies only when a person has been arrested and detained for violation of Philippine criminal laws. It does not apply to extradition proceedings, because extradition courts do not render judgments of conviction or acquittal. Moreover, the constitutional right to bail "flows from the presumption of innocence in favor of every accused who should not be subjected to the loss of freedom as thereafter he would be entitled to acquittal, unless his guilt be proved beyond reasonable doubt." It follows that the constitutional provision on bail will not apply to a case like extradition, where the presumption of innocence is not at issue. That the offenses for which Jimenez is sought to be extradited are bailable in the United States is not an argument to grant him one in the present case. To stress, extradition proceedings are separate and distinct from the trial for the offenses for which he is charged. He should apply for bail before the courts trying the criminal cases against him, not before the extradition court. There is an exception to the No Bail Rule to best serve the ends of justice. After a potential extraditee has been arrested or placed under the custody of the law, bail may be applied for and granted as an exception, only upon a clear and convincing showing (1) that, once granted bail, the applicant will not be a flight risk or a danger to the community; and (2) that there exist special, humanitarian and compelling circumstances including, as a matter of reciprocity, those cited by the highest court in the requesting state when it grants provisional liberty in extradition cases therein. Jimenez claims that he is not a flight risk. To support this claim, he stresses that he learned of the extradition request in June 1999; yet, he has not fled the country. True, he has not actually fled during the preliminary stages of the request for his extradition. Yet, this

fact cannot be taken to mean that he will not flee as the process moves forward to its conclusion, as he hears the footsteps of the requesting government inching closer and closer. That he has not yet fled from the Philippines cannot be taken to mean that he will stand his ground and still be within reach of our government if and when it matters; that is, upon the resolution of the Petition for Extradition. In extradition cases, bail is not a matter of right; it is subject to judicial discretion in the context of the peculiar facts of each case. WHEREFORE, the Petition is GRANTED. The assailed RTC Order dated May 23, 2001 is hereby declared NULL and VOID, while the challenged Order dated July 3, 2001 is SET ASIDE insofar as it granted bail to Respondent Mark Jimenez. The bail bond posted by private respondent is CANCELLED. The Regional Trial Court of Manila is directed to conduct the extradition proceedings before it, with all deliberate speed pursuant to the spirit and the letter of our Extradition Treaty with the United States as well as our Extradition Law. No costs. SO ORDERED.

GR. no. 153675 April 19 2007 Gov. of Hong ong vs Olalia Facts : On January 30, 1995, the Republic of the Philippines and the then British Crown Colony of Hong Kong signed an extradition treaty which took effect on June 20, 1997. On July 1, 1997, Hong Kong reverted back to the Peoples Republic of China and became the Hong Kong Special Administrative Region. Muoz was charged before the Hong Kong Court with three (3) counts of the offense of "accepting an advantage as agent," in violation of Section 9 (1) (a) of the Prevention of Bribery Ordinance, Cap. 201 of Hong Kong. He also faces seven (7) counts of the offense of conspiracy to defraud, penalized by the common law of Hong Kong. On August 23, 1997 and October 25, 1999, warrants of arrest were issued against him. If convicted, he faces a jail term of seven (7) to fourteen (14) years for each charge. On September 13, 1999, the DOJ received from the Hong Kong Department of Justice a request for the provisional arrest of private respondent. The DOJ then forwarded the request to the National Bureau of Investigation (NBI) which, in turn, filed with the RTC of Manila, Branch 19 an application for the provisional arrest of private respondent. On September 23, 1999, the RTC, Branch 19, Manila issued an Order of Arrest against private respondent. That same day, the NBI agents arrested and detained him. On October 14, 1999, private respondent filed with the Court of Appeals a petition for certiorari, prohibition and mandamus with application for preliminary mandatory injunction and/or writ of habeas corpus questioning the validity of the Order of Arrest. On November 9, 1999, the Court of Appeals rendered its Decision declaring the Order of Arrest void. On November 12, 1999, the DOJ filed with this Court a petition for review on certiorari, docketed as G.R. No. 140520, praying that the Decision of the Court of Appeals be reversed. On December 18, 2000, this Court rendered a Decision granting the petition of the DOJ and sustaining the validity of the Order of Arrest against private respondent. The Decision became final and executory on April 10, 2001. Meanwhile, as early as November 22, 1999, petitioner Hong Kong Special Administrative Region filed with the RTC of Manila a petition for the extradition of private respondent, docketed as Civil Case No. 99-95733, raffled off to Branch 10, presided by Judge Ricardo Bernardo, Jr. Judge Bernado denied Munoz petition for bail. On October 22, 2001, Judge Bernardo, Jr. inhibited himself from further hearing Civil Case No. 99-95733. It was then raffled off to Branch 8 presided by judge Olalia. On October 30, 2001, Judge Olalia granted the petition for bail.

On December 21, 2001, the government of Hongkong represented by the DOJ filed an urgent motion to vacate the order on the petition for bail, but it was denied by respondent judge in his Order dated April 10, 2002. Issue: Is bail available in a extradition case? Ruling: The Philippines, along with the other members of the family of nations, committed to uphold the fundamental human rights as well as value the worth and dignity of every person. This commitment is enshrined in Section II, Article II of our Constitution which provides: "The State values the dignity of every human person and guarantees full respect for human rights." The Philippines, therefore, has the responsibility of protecting and promoting the right of every person to liberty and due process, ensuring that those detained or arrested can participate in the proceedings before a court, to enable it to decide without delay on the legality of the detention and order their release if justified. In other words, the Philippine authorities are under obligation to make available to every person under detention such remedies which safeguard their fundamental right to liberty. These remedies include the right to be admitted to bail. While this Court in Purganan limited the exercise of the right to bail to criminal proceedings, however, in light of the various international treaties giving recognition and protection to human rights, particularly the right to life and liberty, a reexamination of this Courts ruling in Purganan is in order. First, we note that the exercise of the States power to deprive an individual of his liberty is not necessarily limited to criminal proceedings. Respondents in administrative proceedings, such as deportation and quarantine,4 have likewise been detained. Second, to limit bail to criminal proceedings would be to close our eyes to our jurisprudential history. Philippine jurisprudence has not limited the exercise of the right to bail to criminal proceedings only. This Court has admitted to bail persons who are not involved in criminal proceedings. In fact, bail has been allowed in this jurisdiction to persons in detention during the pendency of administrative proceedings, taking into cognizance the obligation of the Philippines under international conventions to uphold human rights. The 1909 case of US v. Go-Sioco5 is illustrative. In this case, a Chinese facing deportation for failure to secure the necessary certificate of registration was granted bail pending his appeal. After noting that the prospective deportee had committed no crime, the Court opined that "To refuse him bail is to treat him as a person who has committed the most serious crime known to law;" and that while deportation is not a criminal proceeding, some of the machinery used "is the machinery of criminal law." Thus, the provisions relating to bail was applied to deportation proceedings. In Mejoff v. Director of Prisons6 and Chirskoff v. Commission of Immigration,7 this Court ruled that foreign nationals against whom no formal criminal charges have been filed may be released on bail pending the finality of an order of deportation. As previously stated, the Court in Mejoff relied upon the Universal declaration of Human Rights in sustaining the detainees right to bail. If bail can be granted in deportation cases, we see no justification why it should not also be allowed in extradition cases. Likewise, considering that the Universal Declaration of Human Rights applies to deportation cases, there is no reason why it cannot be invoked in extradition cases. After all, both are administrative proceedings where the innocence or guilt of the person detained is not in issue. Clearly, the right of a prospective extraditee to apply for bail in this jurisdiction must be viewed in the light of the various treaty obligations of the Philippines concerning respect for the promotion and protection of human rights. Under these treaties, the presumption lies in favor of human liberty. Thus, the Philippines should see to it that the right to liberty of every individual is not impaired. Section 2(a) of Presidential Decree (P.D.) No. 1069 (The Philippine Extradition Law) defines "extradition" as "the removal of an accused from the Philippines with the object of placing

him at the disposal of foreign authorities to enable the requesting state or government to hold him in connection with any criminal investigation directed against him or the execution of a penalty imposed on him under the penal or criminal law of the requesting state or government." Extradition has thus been characterized as the right of a foreign power, created by treaty, to demand the surrender of one accused or convicted of a crime within its territorial jurisdiction, and the correlative duty of the other state to surrender him to the demanding state.8 It is not a criminal proceeding.9 Even if the potential extraditee is a criminal, an extradition proceeding is not by its nature criminal, for it is not punishment for a crime, even though such punishment may follow extradition.10 It is sui generis, tracing its existence wholly to treaty obligations between different nations. 11 It is not a trial to determine the guilt or innocence of the potential extraditee.12 Nor is it a full-blown civil action, but one that is merely administrative in character.13 Its object is to prevent the escape of a person accused or convicted of a crime and to secure his return to the state from which he fled, for the purpose of trial or punishment.14 But while extradition is not a criminal proceeding, it is characterized by the following: (a) it entails a deprivation of liberty on the part of the potential extraditee and (b) the means employed to attain the purpose of extradition is also "the machinery of criminal law." This is shown by Section 6 of P.D. No. 1069 (The Philippine Extradition Law) which mandates the "immediate arrest and temporary detention of the accused" if such "will best serve the interest of justice." We further note that Section 20 allows the requesting state "in case of urgency" to ask for the "provisional arrest of the accused, pending receipt of the request for extradition;" and that release from provisional arrest "shall not prejudice re-arrest and extradition of the accused if a request for extradition is received subsequently." A potential extraditee may be subjected to arrest, to a prolonged restraint of liberty, and forced to transfer to the demanding state following the proceedings. "Temporary detention" may be a necessary step in the process of extradition, but the length of time of the detention should be reasonable. The accused had been detained for over two (2) years without having been convicted of any crime. By any standard, such an extended period of detention is a serious deprivation of his fundamental right to liberty. In fact, it was this prolonged deprivation of liberty which prompted the extradition court to grant him bail. While our extradition law does not provide for the grant of bail to an extraditee, however, there is no provision prohibiting him or her from filing a motion for bail, a right to due process under the Constitution. The applicable standard of due process, however, should not be the same as that in criminal proceedings. In the latter, the standard of due process is premised on the presumption of innocence of the accused. As Purganan correctly points out, it is from this major premise that the ancillary presumption in favor of admitting to bail arises. Bearing in mind the purpose of extradition proceedings, the premise behind the issuance of the arrest warrant and the "temporary detention" is the possibility of flight of the potential extraditee. This is based on the assumption that such extraditee is a fugitive from justice. 15 Given the foregoing, the prospective extraditee thus bears the onus probandi of showing that he or she is not a flight risk and should be granted bail. The time-honored principle of pacta sunt servanda demands that the Philippines honor its obligations under the Extradition Treaty it entered into with the Hong Kong Special Administrative Region. Failure to comply with these obligations is a setback in our foreign relations and defeats the purpose of extradition. However, it does not necessarily mean that in keeping with its treaty obligations, the Philippines should diminish a potential extraditees rights to life, liberty, and due process. More so, where these rights are guaranteed, not only by our Constitution, but also by international conventions, to which the Philippines is a party. We should not, therefore, deprive an extraditee of his right to apply for bail, provided that a certain standard for the grant is satisfactorily met.

An extradition proceeding being sui generis, the standard of proof required in granting or denying bail can neither be the proof beyond reasonable doubt in criminal cases nor the standard of proof of preponderance of evidence in civil cases. While administrative in character, the standard of substantial evidence used in administrative cases cannot likewise apply given the object of extradition law which is to prevent the prospective extraditee from fleeing our jurisdiction. In his Separate Opinion in Purganan, then Associate Justice, now Chief Justice Reynato S. Puno, proposed that a new standard which he termed "clear and convincing evidence" should be used in granting bail in extradition cases. According to him, this standard should be lower than proof beyond reasonable doubt but higher than preponderance of evidence. The potential extraditee must prove by "clear and convincing evidence" that he is not a flight risk and will abide with all the orders and processes of the extradition court. In this case, there is no showing that private respondent presented evidence to show that he is not a flight risk. Consequently, this case should be remanded to the trial court to determine whether private respondent may be granted bail on the basis of "clear and convincing evidence." WHEREFORE, we DISMISS the petition. This case is REMANDED to the trial court to determine whether private respondent is entitled to bail on the basis of "clear and convincing evidence." If not, the trial court should order the cancellation of his bail bond and his immediate detention; and thereafter, conduct the extradition proceedings with dispatch. Basco vs Rapatalo [A.M. No. RTJ-96-1335. March 5, 1997 Facts: The complaint dated August 14, 1995, complainant Inocencio Basco charged respondent Judge Leo M. Rapatalo of RTC, Branch 32, Agoo, La Union with gross ignorance or willful disregard of established rule of law for granting bail to an accused in a murder case (Criminal Case No. 2927) without receiving evidence and conducting a hearing. Complainant, who is the father of the victim, alleged that an information for murder was filed against a certain Roger Morente, one of three accused. The accused Morente filed a petition for bail. The hearing for said petition was set for May 31, 1995 by petitioner but was not heard since the respondent Judge was then on leave. It was reset to June 8, 1995 but on said date, respondent Judge reset it to June 22, 1995. The hearing for June 22, 1995, however, did not materialize. Instead, the accused was arraigned and trial was set. Again, the petition for bail was not heard on said date as the prosecution's witnesses in connection with said petition were not notified. Another attempt was made to reset the hearing to July 17, 1995. In the meantime, complainant allegedly saw the accused in Rosario, La Union on July 3, 1995. He later learned that the accused was out on bail despite the fact that the petition had not been heard at all. Upon investigation, complainant discovered that bail had been granted and a release order dated June 29, 1995 [1] was issued on the basis of a marginal note[2] dated June 22, 1995, at the bottom of the bail petition by Assistant Prosecutor Manuel Oliva which stated: "No objection: P80,000.00," signed and approved by the assistant prosecutor and eventually by respondent Judge. Note that there was already a release order dated June 29, 1995 on the basis of the marginal note of the Assistant Prosecutor dated June 22, 1995 (when the hearing of the petition for bail was aborted and instead arraignment took place) when another hearing was scheduled for July 17, 1995. Respondent Judge alleged that he granted the petition based on the prosecutor's option not to oppose the petition as well as the latter's recommendation setting the bailbond in the amount of P80,000.00. He averred that when the prosecution chose not to oppose the petition for bail, he had the discretion on whether to approve it or not. He further declared that when he approved the petition, he had a right to presume that the prosecutor knew what he was doing since he was more familiar with the case, having conducted the preliminary investigation. Furthermore, the private prosecutor was not around at the time the public prosecutor recommended bail. Respondent Judge stated that in any case, the bailbond posted by accused was cancelled and a warrant for his arrest was issued on account of complainant's motion for reconsideration. The Assistant Provincial Prosecutor

apparently conformed to and approved the motion for reconsideration. [3] To date, accused is confined at the La Union Provincial Jail. Issue: Is the judge allowed to grant bail without conducting a hearing to afford the prosecution the opportunity to present evidence that the guilt of the accused is strong? Held: Petition is granted. Respondent Judge was REPRIMANDED with the WARNING . It is in view of the abovementioned practical function of bail that it is not a matter of right in cases where the person is charged with a capital offense punishable by death, reclusion perpetua or life imprisonment. Article 114, section 7 of the Rules of Court, as amended, states, Xxx "No person charged with a capital offense, or an offense punishable by reclusion perpetua or life imprisonment when the evidence of guilt is strong, shall be admitted to bail regardless of the stage of the criminal action." Xxx When the grant of bail is discretionary, the prosecution has the burden of showing that the evidence of guilt against the accused is strong. However, the determination of whether or not the evidence of guilt is strong, being a matter of judicial discretion, remains with the judge. "This discretion by the very nature of things, may rightly be exercised only after the evidence is submitted to the court at the hearing. Since the discretion is directed to the weight of the evidence and since evidence cannot properly be weighed if not duly exhibited or produced before the court,[7] it is obvious that a proper exercise of judicial discretion requires that the evidence of guilt be submitted to the court, the petitioner having the right of cross examination and to introduce his own evidence in rebuttal. Topic: Sec 8, Rule 114 Importance of Hearing JOCELYN V. GRAGEDA, vs. JUDGE NIETO T. TRESVALLES [A.M. MTJ No. 04-1526 : February 02, 2004

FACTS: The instant administrative case arose when Jocelyn V. Grageda filed an Affidavit-Complaint dated January 18, 2000 charging Judge Nieto T. Tresvalles, Municipal Trial Court, Virac, Catanduanes, with gross ignorance of the law and abuse of authority relative to Criminal Case No. 5307 entitled People v. Bernardo Tablizo, Jr. for murder. The respondent judge granted bail to the accused in Criminal Case No. 5307 without the requisite bail hearing, despite the fact that there was an eyewitness to the murder who made a positive identification of the accused. The complainant also alleged that the amount of P30,000.00 printed on the Warrant of Arrest issued by the respondent judge appeared to be snowpaked, an indication that another entry was previously made, possibly a no bail recommendation. The respondent admitted that no bail hearing was conducted in Criminal Case No. 5307, but reasoned that the evidence of the guilt of the accused was not strong. According to the respondent, the matter of granting bail is an exercise of judgment, and that the accused should not be denied his constitutional right to bail. ISSUE: Whether or not the respondent judge committed a grave abuse of discretion. RULING: We agree that the respondent judge is administratively liable for granting bail to an accused charged with murder without conducting the requisite bail hearing.

The importance of a hearing in applications for bail should once more be emphasized. Section 8, Rule 114 provides as follows: Sec. 8. Burden of proof in bail application. - At the hearing of an application for bail filed by a person who is in custody for the commission of an offense punishable by death, reclusion perpetua, or life imprisonment, the prosecution has the burden of showing that the evidence of guilt is strong. The evidence presented during the bail hearing shall be considered automatically reproduced at the trial but, upon motion of either party, the court may recall any witness for additional examination unless the latter is dead, or otherwise, unable to testify. The importance of the Rule lies on the fact that on the result of the bail hearing depends the right of an accused to provisional liberty vis--vis the duty of the State to protect the people against dangerous elements. The resolution of the issue affects important norms in our society: liberty on one hand, and order on the other. To minimize, if not eliminate, error and arbitrariness in a judges decision, the Rules require the judge to hear the parties and then make an intelligent assessment of their evidence. The respondents argument that a hearing is only necessary if there is an application for admission to bail is erroneous. As found by the Executive Judge: [T]he fact that the accused has not even filed yet any application for bail at the time bail was fixed on December 5, 2000 aggravates matters. To state the obvious, there was no occasion for the respondent Judge to exercise any discretion on the matter of bail at that point in time as the accused was not asking to be released on temporary liberty. The respondent Judge should have followed the straight and trodden path, well-traveled by members of the bench, that bail should not be allowed in cases of murder. It might also be worth mentioning, in passing, that the right to bail may be waived considering its personal nature. It arises from the time one is placed in the custody of the law. The fact that the respondent Judge already granted bail when the accused has not been arrested yet compounds the aggravation. Admission to bail presupposes the exercise thereof in accordance with law and guided by the applicable legal principles. The prosecution must first be accorded an opportunity to present evidence because by the very nature of deciding applications for bail, it is on the basis of such evidence that judicial discretion is weighed against in determining whether the guilt of the accused is strong. In other words, discretion must be exercised regularly, legally, and within the confines of due process, that is, after the evaluation of the evidence submitted by the prosecution. In this case, the respondent judge motu proprio granted bail to the accused. The prosecution was not even afforded an opportunity to present its evidence, in accordance with the Rules. We likewise agree with the finding of the Executive Judge that the respondents claim of good faith will not exonerate him from administrative liability. The respondent Judge also argues in his Position Paper submitted to the undersigned Executive Judge that there is absolutely no evidence to show that he was motivated by bad faith, fraud, dishonesty or corruption in granting bail. As such, he argues that his act which was done in his official capacity is not subject to disciplinary action. Unfortunately for the respondent Judge, it is already settled that when a judge grants bail to a person charged with a capital offense, or an offense punishable by reclusion perpetua or life imprisonment without conducting the required [bail] hearing, he is considered guilty of ignorance or incompetence the gravity of which cannot be excused by a claim of good faith or excusable negligence. Section 26 cases: RULE 114 BAIL SEC. 26. Bail not a bar to objections on illegal arrest, lack of or irregular preliminary investigation. An application for or admission to bail shall not bar the accused from challenging the validity of his arrest or the legality of the warrant issued therefore, or from assailing the regularity or questioning the absence of a preliminary investigation of the charge against him, provided that he raises them before entering his plea. The court shall

resolve the matter as early as practicable but not later than the start of the trial of the case. (n) PEOPLE V. EJANDRA G.R. No. 134203, May 27, 2004 FACTS OF THE CASE: On or about July 2, 1997, in Quezon City, Elvie Ejandra, Magdalena Calunod, Edwin Tampos and Roel Revilla , while confederating, conniving, conspiring, and helping each and one another, did then and there with the use of force, threat and intimidation, take and carry away, a nine-year-old minor child, Ed Henderson Tan, against the will and consent of the latter nor any of his natural and legal parents or guardian, to an unknown destination, detain, hold and control Ed Henderson Tan depriving him of his liberty, and during their control and custody of Ed Henderson Tan, call, demand and negotiate the payment of ransom money from Eddie Tan, the father of Ed Henderson Tan, for the safe release and return of the victim Ed Henderson, otherwise, the victim would be harmed or killed, the victims father Eddie Tan actually paid the accused the amount of P548,000.00 as ransom money, for the safe release of the victim to the damage and prejudice of the victim Ed Henderson Tan and his father Eddie Tan. This is a review on automatic appeal of the Decision of the Quezon City Regional Trial Court, Branch 219, convicting the above-named accused of kidnapping for ransom, and sentencing them to suffer the death penalty. In an appeal to the Supreme Court, Ejandra and Calunod asserted that they were arrested without any warrant therefor. ISSUE: Whether or not the accused can still challenge the validity of the arrest upon appeal of the case to the Supreme Court. RULING OF THE COURT: NO. The Office of the Solicitor General submits that in failing to assail any irregularity in their arrest before they were arraigned for the crime charged, the appellants thereby waived their right to do so. The appellants even failed to file their respective ucounter-affidavits during the preliminary investigation of the charge against them at the Department of Justice. Moreover, the prosecution adduced overwhelming evidence to prove the crime charged that the appellants were the perpetrators of the said crime. We agree with the Office of the Solicitor General that the appellants Ejandra and Calunod waived any irregularities relating to their warrantless arrest when they failed to file a motion to quash the Information on that ground, or to object to any irregularity in their arrest before they were arraigned. They are now estopped from questioning the legality of their arrest. Yusop vs Sandiganbayan Facts: Acting on an Affidavit-Complaint3 filed by a certain Erlinda Fadri, the Office of the Ombudsman-Mindanao issued an Order4 dated September 19, 1995, naming the following as respondents: benjamin Arao, Fredireck Winters, Pelaez Pantaran, Eduardo Dablo, Efren Sissay and the city jail warden of Pagadian City. The Order also required respondents, within

ten days from receipt thereof, to submit their counter-affidavits and other pieces of controverting evidence. The Office of the Ombudsman for Mindanao issued a Resolution dated January 15, 1998,5 recommending the prosecution of "the aforenamed respondents" for violation of Article 269 of the Revised Penal Code and Section 3-a in relation to Section 3-e of Republic Act No. 3019 as amended. Significantly, the name of Petitioner Alvarez A. Yusop was included as one of the persons to be prosecuted, although he was not one of the original respondents mentioned in the Order of September 19, 1995. Ombudsman Aniano A. Desierto approved the recommendation. Accordingly, two Informations were filed with the Sandiganbayan. They were docketed as Criminal Case Nos. 24524 (violation of Section 3-a of RA 3019) and 24525 (unlawful arrest under Article 269 of the Revised Penal Code). On April 16, 1998, an Order of Arrest was issued by the Sandiganbayan in Criminal Case No. 24524. Petitioner, however, posted a bail bond before the Regional Trial Court of Dipolog City on May 20 of the same year. On the same day, he filed a "Motion To Remand Case To The Ombudsman - Mindanao For Preliminary Investigation." In Resolution dated June 8, 1998, the Sandiganbayan denied the Motion of petitioner for his alleged failure to submit himself to the jurisdiction of the anti-graft court. On August 8, 1998, petitioner filed a Motion to Dismiss, grounded again on the lack of preliminary investigation. In an Order dated September 22, 1998, the Sandiganbayan resolved not to take action on the Motion, because petitioner had not yet submitted himself to its jurisdiction insofar as Criminal Case No. 24525 was concerned. On the scheduled arraignment on February 15, 1999, petitioner reiterated his claim that he had not been accorded preliminary investigation. In its two assailed Orders, the Sandigabayan rejected his claim and proceeded with the arraignment. Hence, this recourse.6 Issue : 1. Whether the Sandiganbayan, despite being informed of the lack of preliminary investigation with respect to petitioner, In Criminal Case No. 24524, committed grave abuse of discretion in proceeding with his arraignment? 2. Is the filing of the bail bond constitute a waiver for seeking the right of preliminary investigation? Held: Petition is partly meritorious remanded in Sandiganbayan for conduct of Preliminary investigation. Annent to the first issue. We disagree with the Sandiganbayan's reliance on Section 27 of Republic Act 6770.18 This provision cannot justify the evasion of the requirement set forth in the Rules of Court for conducting preliminary investigation. The law does not sanction such interpretation, for it deals merely with the finality of orders, directives and decisions of the Office of the Ombudsman -- not the deprivation of the substantive right to a preliminary investigation. Moreover, petitioner cannot be bound by the Ombudsman's January 15, 1998 Resolution, which recommended the filing of charges. He was not a party to the case and was not accorded any right to present evidence on his behalf. In any event, even the Ombudsman agrees that petitioner was deprived of this right and believes that the basic rudiments of due process are complied with." 19 For its part, the Sandiganbayan opted to remain silent when asked by this Court to comment on the Petition. Anent to the second issue: In Go v. Court of Appeals,16 this Court held that "the right to preliminary investigation is waived when the accused fails to invoke it before or at the time of entering a plea at arraignment." Conversely, if the accused does invoke it before arraignment, as the petitioner did in this case, the right is not waived. Neither did the filing of a bail bond constitute a waiver of petitioner's right to preliminary investigation. Under Section 26, Rule 114 of the Revised Rules of Criminal Procedure, "[a]n application for or admission to bail; shall not bar the accused from challenging the validity of his arrest or the legality of the warrant issued therefor, or from assailing the

regularity or questioning the absence of a preliminary investigation of the charge against him, provided that he raises them before entering his plea. xxx." We stress that the right to preliminary investigation is substantive, not merely formal or technical. To deny it to petitioner would deprive him of the full measure of his right to due process.17 Hence, preliminary investigation with regard to him must be conducted. RULE 116 ARRAIGNMENT AND PLEA PEOPLE OF THE PHILIPPINES, plaintiff-appellee vs. EDUARDO AGBAYANI y MENDOZA, accused-appellant. [G.R. No. 122770. January 16, 1998] FACTS OF THE CASE: Before this Court on automatic review is the decision of the Regional Trial Court of Quezon City, Branch 106, in view of the death penalty imposed by it for the crime of rape, defined and penalized under Article 335 of the Revised Penal Code, as amended by R.A. 7659. On 12 September 1994, the Station Investigation and Intelligence Division of the National Capital Region Command, Philippine National Police (PNP), endorsed to the Office of the City Prosecutor of Quezon City the complaint of Eden Agbayani (hereafter EDEN) for rape against her father, herein accused-appellant Eduardo Agbayani y. Mendoza. After appropriate preliminary investigation, a complaint for rape signed by EDEN, assisted by her sister Fedelina Agbayani, and subscribed and sworn to before Asst. City Prosecutor Charito B. Gonzales, was filed against appellant with the Regional Trial Court of Quezon City on 27 October, 1994. The case was docketed as Criminal Case No. Q-94-59149, then set for arraignment, pre-trial and trial on 22 December 1994. At his arraignment on 22 December 1994, appellant, assisted by Attys. Samuel Baldado and Edwin dela Cruz as counsel de oficio, entered a plea of not guilty. Upon agreement of the parties, trial on the merits immediately followed, with the prosecution presenting the first witness, Dr. Florante Baltazar, a Medico-Legal Officer of the PNP Crime Laboratory, who cross-examined by Atty. Baldado. On the succeeding dates of trial, the prosecution presented EDEN and SPO1 Salvador Buenviaje. During these hearings, however, appellant was represented by Atty. Arturo Temanil of the Public Attorneys Office. After the trial proper, the trial court convict the victim and sentence him to death penalty, this record is forwarded to the higher court for automatic review of the case. ISSUE: own choice. RULING OF THE COURT: Has been held to be mandatory and denial of this right is reversible error and a ground for new trial. The courts should comply with Rule 116, Sec. 3. It would be a grievous error to proceed by sentencing the accused without due process of law and this is not complete, when the accused is denied the right recognized by said rule. The records must show compliance therewith or that the accused renounced his right to be assisted by counsel. These is demanded by the interest of justice and remove all doubt that if the accused had Whether or not the petitioner was apprise of his right to have counsel of his

waived said right, he was fully informed before giving his plea of its consequences. Omission by courts whether voluntary should not truly be censured but also condemned. Since appellant has miserably failed to show that he was not informed of his right to counsel, the presumptions that the law has been obeyed and official duty has been regularly performed by the trial court stand. In other words, the trial court is presumed to have complied with its four-fold duties under Section 6 of Rule 116 of the Rules of Court, namely, (1) to inform the accused that he has the right to have his own counsel before being arraigned; (2) after giving such information, to ask accused whether he desires the aid of counsel; (3) if he so desires to procure the services of counsel, the court must grant him reasonable time to do so; and (4) if he so desires to have counsel but is unable to employ one, the court must assign counsel de oficio to defend him. It is settled that the failure of the record to disclose affirmatively that the trial judge advised the accused of his right to counsel is not sufficient ground to reverse conviction. The reason being that the trial court must be presumed to have complied with the procedure prescribed by law for the hearing and trial of cases, and that such a presumption can only be overcome by an affirmative showing to the contrary. Thus it has been held that unless the contrary appears in the record, or that it is positively proved that the trial court failed to inform the accused of his right to counsel, it will be presumed that the accused was informed by the court of such right. WHEREFORE, judgment is hereby rendered AFFIRMING the decision of the Regional Trial Court of Quezon City, Branch 106, in Criminal Case No. Q-94-59149 finding accusedappellant EDUARDO AGBAYANI y MENDOZA guilty beyond reasonable doubt as principal of the crime of rape defined and penalized under under Article 335 of the Revised Penal Code, as amended by R.A. No. 7659, and imposing upon him the penalty of DEATH, subject to the above modification as to the amount of indemnity. Upon the finality of this Decision, let certified true copies thereof, as well as the records of this case; be forwarded without delay to the Office of the President for possible exercise of executive clemency pursuant to Article 83 of the Revised Penal Code, as amended by Section 25 of R.A. No. 7659. With costs de oficio. SO ORDERED. PEOPLE OF THE PHILIPPINES, appellee, vs. WILLIAM ONG y LI and CHING DE MING @ ROBERT TIU, appellants. (G.R. No. 137348 Facts: On July 27, 1998 accused William Ong y Li and Ching De Ming @ Robert Tiu, both Chinese nationals, were charged with violation of Section 15, Article III, in relation to Section 2, Article I, of Republic Act No. 6425, otherwise known as The Dangerous Drugs Act of 1972, as amended. TheInformation states that the two accused, on July 24, 1998 were busted in an entrapment operation selling or offering for sale for 980.50 grams of shabu Upon arraignment, the two 2 accused pled not guilty. The records do not show whether they had sufficient knowledge of the English language. Their trial proceeded. In the course of the trial, the two 2 accused were given the services of a Chinese interpreter. June 21, 2004)

On November 1998 the trial court convicted the Ong and De Ming and imposed on them the penalty of death. They each were also ordered to pay a fine of P1 million pesos. The 2 accused on the other hand maintained their innocence and averred that there guilt was not proven beyond reasonable doubt. The case was brought to the SC on automatic review. Issue: Was the crime allegedly committed by appellants duly proven and thus warrant the penalty of death? Ruling: Rule 116, Section 1 (a) of the Revised Rules of Criminal Procedure, as amended, provides: SEC. 1. Arraignment and plea; how made.(a) The accused must be arraigned before the court where the complaint or information was filed or assigned for trial. The arraignment shall be made in open court by the judge or clerk by furnishing the accused with a copy of the complaint or information, reading the same in the language or dialect known to him, and asking him whether he pleads guilty or not guilty. The prosecution may call at the trial witnesses other than those named in the complaint or information. The arraignment of appellants violates the above rule. Appellants are Chinese nationals. Their Certificate of Arraignment states that they were informed of the accusations against them. It does not, however, indicate whether the Information was read in the language or dialect known to them. What leaps from the records of the case is the inability of appellants to fully or sufficiently comprehend any other language than Chinese and any of its dialect. Despite this inability, however, the appellants were arraigned on an Information written in the English language. The requirement that the information should be read in a language or dialect known to the accused is mandatory. It must be strictly complied with as it is intended to protect the constitutional right of the accused to be informed of the nature and cause of the accusation against him. The constitutional protection is part of due process. Failure to observe the rules necessarily nullifies the arraignment. More important than the invalid arraignment of the appellants, the evidence by the prosecution failed to prove that appellants willfully and unlawfully sold or offered to sell shabu. Appellants conviction is based on the lone testimony of SPO1 Gonzales. He was the designated poseur-buyer in the team formed for the buy-bust operation. But records reveal that he was not privy to the sale transaction that transpired between the CI and appellant William Ong, the alleged pusher. Also the claim of entrapment by the police officers failed to convince that indeed there was a transaction that took place. The prosecution evidence about the buy-bust operation is incomplete. The confidential informant who had sole knowledge of how the alleged illegal sale of shabu started and how it was perfected was not presented as a witness. His

testimony was given instead by SPO1 Gonzales who had no personal knowledge of the same. WHEREFORE, the Decision of the court a quo is REVERSED and SET ASIDE. Appellants WILLIAM ONG y LI and CHING DE MING @ ROBERT TIU, are ACQUITTED G.R. No. 106695 August 4, 1994 EDWARD T. MARCELO, DIONILO D. MARFIL, CELIA C. CABURNAY, and DANIEL T. PASCUAL, petitioners, vs. THE COURT OF APPEALS, THE PEOPLE OF THE PHILIPPINES, HON. PEDRO T. SANTIAGO, in his capacity as The Presiding Judge of the Regional Trial Court of Quezon City, Branch 101, and THE QUEZON CITY PROSECUTOR, respondents. FACTS: In a complaint-affidavit sworn to on 18 March 1991 and filed with the Office of the City Prosecutor of Quezon City, Jose T. Marcelo charged the petitioners with falsification of public documents committed by forging the signature of Jose P. Marcelo, Sr. in six voting trust agreements (VTA's). Submitted in support of the affidavit-complaint were the findings of the National Bureau of Investigation (NBI) and of the PC/PNP Crime Laboratory that the signature on the VTA's purporting to be that of Jose P. Marcelo, Sr. and the specimen or standard signature of the latter were not written by one and the same person. After conducting a preliminary investigation, Assistant City Prosecutor Domingo Israel found "more than sufficient evidence" of the forgery of the signature of Jose P. Marcelo, Sr., as "found and concluded by two (2) national police agencies, the NBI and PCCL," and recommended the filing of the case in court. On 30 May 1991, an information for falsification of public documents was filed with the Regional Trial Court (RTC) of Quezon City, docketed as Criminal Case No. Q-91-21285, and raffled to Branch 96 5 thereof (hereinafter Bersamin court). On 13 June 1991, the petitioners, in a Manifestation and Motion, informed the Bersamin court of the filing of their Motion for Review and prayed that further proceedings in the case be suspended until the resolution of the Motion for Review. Then, on 27 August 1991, the petitioners filed an urgent motion to defer the arraignment on 28 August 1991 until the resolution of their Motion for Review. 12 Acting thereon, and over the vigorous opposition of the private prosecutor, Judge Bersamin issued an order on 28 August 1991 13 resetting the arraignment to 8 October 1991 and directing the City Prosecutor of Quezon City "to conclude the pending review of the resolution of the filing Prosecutor Domingo Israel and to render a report of the results of the review on or before" 8 October 1991. On 15 November 1991, the Review Committee handed down a resolution, 16 approved by Acting City Prosecutor Lydia Navarro on 29 November 1991, recommending the reversal of the Israel resolution and the withdrawal of the information in Criminal Case No. Q-91-21285. Then on 5 December 1991, the petitioners filed a Manifestation and Motion informing the Bersamin court of the reversal and praying for the dismissal of the case. 17 This was followed on 10 December 1991 by the motion of Assistant City Prosecutor Conrado M. Jamolin which prayed for the withdrawal of the information in Criminal Case No. Q-91-21285 because of the resolution of the review committee. 18 The private prosecutor opposed this motion. On 13 December 1991, Judge Bersamin, agreeing with the findings and conclusions of the Review Committee, issued an order granting the motion to dismiss of the accused and motion to withdraw information of the public prosecutor. However, on 27 January 1992, then Secretary of Justice Silvestre R. Bello III handed down a resolution granting the complainant's appeal, reversing the 15 November 1991 Resolution of the Review Committee, and ordering the filing of a new information. The new information, 23 signed by Assistant City Prosecutor Ralph Lee, was filed on 5 February 1992 pursuant to the resolution of Secretary Bello, docketed as Criminal Case No. Q-92-28104, and then raffled to Branch 101 presided over by Judge Pedro Santiago (hereinafter Santiago court) of the RTC of Quezon City. The petitioners posted bail.

On 3 March 1992, the petitioners filed a Motion to Quash the Information on the ground that the dismissal of Criminal Case No. Q-91-21285 was already final and that the appeal subsequently taken by the private prosecutor to and the resolution thereon by the Secretary of Justice are null and void and cannot be a valid basis for any authority to file the new information or for the court to acquire jurisdiction over the case. On 20 March 1992, Judge Santiago issued an order denying the motion to quash on the principal ground that it was not based on any of the grounds enumerated in Section 3, Rule 117 of the Rules of Court. ISSUE: Whether a pre-arraignment dismissal of a criminal case by the trial court, which relied on the reversal by the Review Committee of the Office of the City Prosecutor of the investigating prosecutor's resolution to file the information, bars the filing of a new information for the same offense after the Secretary of Justice reversed the resolution of the review committee. RULING: The petition is without merit. The Court of Appeals correctly dismissed the petitioners' special civil action for certiorari not necessarily for the reason it relied upon, i.e., "certiorari and prohibition are not the correct remedies against an order denying a motion to quash," but because the Santiago court did not act without or in excess of jurisdiction or with grave abuse of discretion in denying the motion to quash. It is settled that if a court, in denying the motion to quash (or a motion to dismiss), acts without or in excess of jurisdiction or with grave abuse of discretion, certiorari or prohibition lies. The denial by the Santiago court of the motion to quash suffers from no fatal infirmity. The petitioners' contention that the prosecutor did not have the authority to file the information because he acted upon an order of the Secretary of Justice which is void in the light of Crespo vs. Mogul 37 is untenable. Nothing in the said ruling forecloses the power or authority of the Secretary of Justice to review resolutions of his subordinates in criminal cases. The Secretary of Justice is only enjoined to refrain as far as practicable from entertaining a petition for review or appeal from the action of the prosecutor once a complaint or information is filed in court. In any case, the grant of a motion to dismiss, which the prosecution may file after the Secretary of Justice reverses an appealed resolution, is subject to the discretion of the court. Pursuant to Section 1 of Department Circular No. 7, only resolutions of the Chief State Prosecutor, the Regional State Prosecutor, and the Provincial or City Prosecutor dismissing a criminal complaint may be appealed to the Secretary of Justice, except as otherwise provided in Section 4 thereof. Under the latter, a resolution of the aforesaid prosecutors finding probable cause may be appealed only upon a showing of manifest error or grave abuse of discretion; however, even with such showing, the appeal shall not be entertained if the appellant had already been arraigned, and if the arraignment took place during the pendency of the appeal, the appeal shall be dismissed motu proprio by the Secretary of Justice. In this case, the petitioners did not at once appeal to the Secretary of Justice from the resolution of Assistant Prosecutor Israel. Instead, they initially filed the Motion for Review. It is clear that the Bersamin court knew and took cognizance of the Motion for Review, deferred the arraignment of the accused until the resolution of the said motion, and even directed the Office of the City Prosecutor "to conclude the pending review . . . and to render a report of the results of the review. In thus recognizing and allowing the Motion for Review, the Bersamin court deferred to the authority of the prosecution arm of the government to resolve with finality the issue of whether or not the information should have been filed. The Review Committee's resolution was of course not final because under Department Circular No. 7 both the offended party and the petitioners could still appeal therefrom to the Secretary of Justice under Section 1 and Section 4 thereof. The Bersamin court knew or was expected to know, since it had to take judicial notice of Department Circular No. 7, that the

resolution of the Review Committee was not final. The offended party had, in fact, appealed from the said resolution to the Secretary of Justice on 10 December 1991. Consequently, the 5 December 1991 Manifestation and Motion of the petitioners praying for the dismissal of the case and the 10 December 1991 motion of Assistant City Prosecutor Jamolin asking for the withdrawal of the information were prematurely filed, because as to the first, the period of the offended party to appeal from the resolution to the Secretary of Justice had not yet lapsed or even begun, there being no showing of the date the offended party received a copy thereof; and, as to the second, an appeal had in fact been filed on 10 December 1991. The withdrawal of the information in Criminal Case No. Q-91- 21285, or even the dismissal of the said case as decreed by the Bersamin court, did not bar the filing of a new information as directed by the Secretary of Justice in his Resolution of 27 January 1992. No jeopardy had attached as a result of the earlier termination of Criminal Case No. Q-91-21285 because the petitioners therein had not been arraigned and had, in fact, asked for its dismissal for a cause other than that which would constitute double jeopardy.The Santiago court, therefore, correctly denied the petitioners' motion to quash in Criminal Case No. Q-92-28104 and the Court of Appeals committed no reversible error in dismissing the petition in CA-G.R. SP No. 27681. G.R. No. 81389 February 21, 1989 PEOPLE OF THE PHILIPPINES, petitioner, vs. HON. RENATO C. DACUDAO, Presiding respondents. FACTS : On August 11, 1987, an Information for Murder with the qualifying circumstances of treachery and evident premeditation was filed before the Regional Trial Court of Cebu, Branch XIV, presided by respondent Judge Renato C. Dacudao, against accused Rey Christopher Paclibar and Nero Desamparado for the death of Cesarlito Nolasco. Upon arraignment, accused Rey Christopher Paclibar entered a plea of 'not guilty' to the offense charged. On September 18, 1987, accused Rey Christopher Paclibar filed a motion for bail, furnishing the Provincial Fiscal of Cebu with a copy thereof. On September 29, 1987, and without conducting a hearing in the application for bail, respondent Judge summarily issued the following Order: ORDER .Considering the motion for bail and the opposition thereto, and, on the basis of the complaint at bar and the sworn statement of Patrolman Elpidio Desquitado, Tadeo Abello and Romeo Torrizo all of the Integrated National Police, Bantayan (Cebu) Police Station, which constitute the essential evidence (so far) of the prosecution in this case, this Court hereby resolves to grant the motion for bail presented by Atty. Bernardito A. Florida and to this end hereby fixes the bailbond for the accused Rey Christopher Paclibar at P50,000.00. From the foregoing Order, private prosecutor Alex R. Monteclar filed a motion for reconsideration alleging that "THE GRANTING OF BAIL TO THE ACCUSED WITHOUT A HEARING IS VIOLATIVE OF PROCEDURAL DUE PROCESS, HENCE. NULL AND VOID ISSUE: Whether or not the granting of bail to the accused without a hearing is violative of Procedural Due Process. RULING: The answer is in the affirmative. We are of the considered opinion that whether the motion for bail of a defendant who is in custody for a capital offense be resolved in a summary proceeding or in the course of a regular trial the prosecution must be given an opportunity to present, within a reasonable time, all the evidence that it may desire to introduce before the court should resolve the motion for bail. If, as in the criminal case involved in the instant special civil action, the prosecution should be denied such an opportunity, there would be a violation of procedural due process, and the order of the court granting bail should be considered void on that ground. Bail is not a matter of right as regards persons charged with offenses punishable by reclusion perpetua when the evidence of guilt is strong. Thus, Sec. 5, Art. 114 of the Rules of Criminal Procedure requires a hearing

before resolving a motion for bail by persons charged with offenses punishable by reclusion perpetua where the prosecution may discharge its burden of showing that the evidence of guilt is strong. The case at bar, which is murder, is punishable by reclusion perpetua. Finally, the defense contends that the Judge did not commit any error because actually the complaint in the Municipal Circuit Trial Court is for homicide only) and the recommended Information was also for homicide. We note, however, that when the same was filed with the Regional Trial Court, it was already an Information for murder. The amendment or changing of an information prior to the plea of the accused is allowed there being no prejudice to him. Thus, in the case of Gaspar v. Sandiganbayan (144 SCRA 415 [1986]), this Court held that, "no actual double jeopardy exists where the petitioner had not yet pleaded guilty to the offense."

You might also like